Sin n последовательность: Предел синуса на бесконечности — Математика всем и для всех — LiveJournal

2

I семестр / Математический Анализ — ответы — 21-31 билеты / вопрос 22

22 Критерий Коши сходимости последовательности.

Определение: Последовательность {xn} называется фундаментальной, если   > 0  N,  n >N и  натурального p: xn+pxn < .

Так как m = n+p — тоже произвольное число >N, то определение фундаментальности можно сформулировать следующим образом:   > 0  N,  n >N и  m > N:xmxn < .

Геометрически фундаментальность последовательности {xn} означает, что для любого сколь угодно малого  существует такой номер N, что любые два члена последовательности с большим, чем N, номерами, отстоят друг от друга не более, чем на .

(здесь рисунок)

Пример: Докажем, что последовательность =1, ,, … — фундаментальная. Зададим произвольное  > 0, возьмем N > . Тогда > , и  n >N и  натурального p: xn+pxn = =-< << . Это и означает, что последовательность — фундаментальная.

Лемма 2. Фундаментальная последовательность ограничена.

Доказательство: Пусть {xn} — фундаментальная последовательность. Возьмем какое-нибудь положительное , например,  = 1. По определению фундаментальности,  N,  n >N и  m > N:xmxn < 1. Зафиксируем какое-нибудь m0 > N, тогда < 1 при n >N, или — 1 < xn < + 1 при n >N. Таким образом, все члены последовательности с номерами n >N лежат в интервале (- 1, + 1), вне этого интервала лежит только конечное число членов последовательности.

Это и означает, что последовательность {xn} ограничена.

Лемма доказана.

Теорема 6.4 (критерий Коши сходимости последовательности) Для того, чтобы числовая последовательность сходилась, необходимо и достаточно, чтобы она была фундаментальной.

Доказательство.

  1. Необходимость. Дано: Последовательность {xn} сходится. Требуется доказать, что последовательность {xn} — фундаментальная. Пусть =a. Зададим произв.  > 0.

По определению предела,  N,  n > N:xna < , и  m > N:xma < . Если n > N, m > N, то x

mxn=(xma) — (xna) +< .

Это и означает по определению, что последовательность {xn} — фундаментальная. Необходимость доказана.

  1. Достаточность. Дано: Последовательность {xn} — фундаментальная. Требуется доказать: {xn} сходится. По лемме 2, последовательность {xn} ограничена, следовательно, можно выделить сходящуюся подпоследовательность . Пусть = a. Докажем, что = a. Зададим произвольное  > 0. Так как подпоследовательность сходится к a, начиная с некоторого номера N1 все члены  {- окрестности точки a}, а так как последовательность {x

    n} — фундаментальная, то начиная с некоторого номера N2 все члены xn отстоят от членов меньше, чем на . Следовательно, начиная с номера N = max (N1, N2) все члены последовательности xn  {- окрестности точки a}, а это и означает, что = a, что и требовалось доказать. Теорема доказана.

[12] Сформулировать определение нефундаментальной последовательности.

Пример: Рассмотрим последовательность {sin n}. Докажем, что эта последовательность расходится. Для этого достаточно доказать, что она не является фундаментальной. Предположим противное: допустим, что {sin n} — фундаментальная. Тогда если   > 0 

N,  n >N и  натурального p: xn+pxn= sin(n+p) — sin n< . Возьмем p = 2.

sin(n+2) — sin n< 2sin 1cos(n+1)< .

  > 0  N,  n >N: cos(n+1)< .

=> {cos n} — бесконечно малая, то есть cos n  0 при n  .

cos(n+1) = cos ncos 1- sin nsin 1.

sin n = ( cos ncos 1 — cos(n+1))  0 при n  .

cos n  и sin n  0 при n  . Но это противоречит тому, что cos2n + sin2n =1.

Полученное противоречие доказывает, что последовательность {sin

n} расходится.

Попробуйте исследовать вопрос, сколько предельных точек у последовательности {sin n}.

исчисление — показывает, что $a_n=\sin(n)$ не сходится

Задать вопрос

спросил

Изменено 6 лет, 6 месяцев назад

Просмотрено 9к раз

$\begingroup$

Показать, что $a_n=\sin(n)$ не сходится


Моя идея:

Возьмем две подпоследовательности: $a_{n_k}=\sin(\frac {\pi k} 2)$ , $a_{n_l}=\sin(\frac {2\pi l} 3 )$

Итак: $\forall n$ : $\lim_{n\to\infty} a_{n_k}=1$, $\lim_{n\to\infty} a_{n_l}=-1$

Таким образом, две бесконечные подпоследовательности сходятся к разным пределам, поэтому последовательность $a_n$ не сходится.

Верно?

Редактировать:

Противоположное определение предела последовательности:

$\exists\epsilon>0 : \forall n\in N : \exists n>N \Rightarrow |x_n-L|>\epsilon$

Возьмем $\epsilon =1$ и мы знаем, что $\sin( n)$ ограничен, поэтому возьмем его супермуму: 1

$|1-1|>\epsilon=1\Rightarrow 0>1 \Rightarrow$ Противоречие.

  • исчисление
  • последовательности-и-ряды
  • пределы
  • проверка-проверка

$\endgroup$

13

$\begingroup$

Ваш аргумент неверен, так как вы вычисляете в нем значения $\sin{k\pi\over 2}$, которые не являются значениями исходной последовательности.

Вот доказательство, не использующее аргументы плотности:

Предположим, что $\lim_{n\to\infty}\sin n=\sigma\in{\mathbb R}$. Затем $$2\cos n\>\sin 1=\sin(n+1)-\sin(n-1)\to 0\qquad(n\to\infty)\ ,$$ откуда следует $\lim_{n\to\infty}\cos n=0$, откуда $\sigma\in\{-1,1\}$. Впуск $n\to\infty$ в $$\sin(n+1)=\sin n\>\cos 1+\cos n\>\sin 1$$ тогда будет означать $\cos 1=1$, что явно неверно.

$\endgroup$

2

$\begingroup$

Здесь нужен более глубокий результат. Обозначим через $\{x\}$ дробную часть $x$. По теореме Кронекера множество $\left\{ \{\frac{n}{2\pi}\}:n\in\Bbb{N}\right\}$ является плотным подмножеством $[0,1] $, (поскольку $\pi$ иррационально.) Итак, существуют две последовательности целых чисел $(n_k)_k$ и $(m_k)_k$ такие, что

$$ \lim_{k\to\infty}\left\{\frac{n_k}{2\pi}\right\}=0,\quad\hbox{and}\quad\lim_{k\to\infty}\left \{\ frac {m_k} {2 \ pi} \ right \} = \ frac {1} {4} $$ Эквивалентно $$ \lim_{k\to\infty}\left(n_k-2\pi\left\lfloor\frac{n_k}{2\pi}\right\rfloor\right)=0,\quad\hbox{and}\quad \lim_{к\к\infty} \left(m_k-2\pi\left\lfloor\frac{m_k}{2\pi}\right\rfloor\right)=\frac{\pi}{2} $$ Это $\lim\limits_{k\to\infty}\sin(n_k)=0$ и $\lim\limits_{k\to\infty}\sin(m_k)=1$. Таким образом, последовательность $(\sin(n))_n$ не сходится. 9n}n$$сходится?

(Задача поставлена ​​22.06.2017 аспирантами Центра им. Г. Штейнхауса Вроцлавской политехники. Обещанный приз за решение — «butelka miodu pitnego», см. стр. 37 тома 1 Львовской шотландской книги. Чтобы получить приз, напишите на адрес электронной почты: [email protected]).

  • nt.теория чисел
  • реальный анализ
  • последовательности-и-ряды
  • диофантовое приближение

$\endgroup$

17

$\begingroup$

Обратите внимание, что если бы $\pi$ было рациональным (с четным числителем), то $\sin(n)$ периодически равнялось бы $1$, поэтому ряд расходился бы. Аналогично, если бы $\pi$ было достаточно сильным числом Лиувилля. Таким образом, для установления сходимости необходимо использовать некоторую количественную меру иррациональности $\pi$.

Добавить комментарий

Ваш адрес email не будет опубликован. Обязательные поля помечены *